PT32.S4.Q24 - increasing complexity of scientific inquiry

Dgelf321Dgelf321 Free Trial Member
edited January 2016 in Logical Reasoning 87 karma
http://7sage.com/lsat_explanations/lsat-32-section-4-question-24/
Can someone please explain this? I got the right answer but I can't justify my reasoning

Comments

  • brna0714brna0714 Alum Inactive ⭐
    1489 karma
    It helps me to the think of Must be True questions as very clearly distinct from Most Strongly Supported questions. In other words, you're not looking for an inference that is supported by the entire stimulus necessary, but rather one that could be supported by just a small portion of the stimulus. My apologies if you already know all of this.

    For this particular question, I believe B is justified because it can be thought of as a restatement of the second sentence in the stimulus. "Usually" can be thought of as "most." In addition, we're talking about the same subset of reports of clinical trials, those "involving patients from several participating hospitals." To say that these reports have multiple authors is equivalent, IMO, of saying they are coauthored by physicians from each establishment. Several "equals" multiple, in this case.

    Each of the other answer choices can be eliminated which is what allowed me to find the correct answer choice the first time through. I didn't stumble upon the justification detailed above until my BR. I'd be happy to explain my reasoning for eliminating the others if you think that would help.
  • Dgelf321Dgelf321 Free Trial Member
    87 karma
    Thanks a lot for the explanation. It was very eloquent and descriptive. My main issue with this question was that I just "felt" like the answer is B. Like J.Y. said, that's great on the day of the test -but there's no guarantee I'm gonna "feel" the correct answer on test day. I vacillated between B and D and eventually went with the former. Can you tell me why you ruled out D? Is it because it's basically the inverse ( or incorrect reversal/wrong reversal as many test prep companies call it) ? Thanks again!
  • brna0714brna0714 Alum Inactive ⭐
    edited March 2015 1489 karma
    I was able to eliminate answer choice D because, based on the stimulus alone, we know very, very little about "physics papers authored by researchers from multiple laboratories." I can see why D might be tempting but I believe it be comes more clear when you consider that the answer choice only deals with physics papers, and because of this, we only really need to compare the last sentence of the stimulus to the information presented in D.

    In that last sentence, we're told that physics papers (more specifically those that are "reporting results using subsystems developed at multiple laboratories") generally have authors from each of those laboratories. That's a fairly small subset of physics papers. In order for D to be correct, we would need information on another, more general group of physics papers, those authored by researchers from multiple laboratories. This is a different group from the one mentioned above and we don't know anything about that group as a whole. It could be that they “usually report results from experiments using subsystems developed at each laboratory,” as stated in D, or it could be that they usually get frustrated, give up half way through and go party instead. The point here is that D does not have to be true. We could say, “it is not the case that physics papers authored by researchers from multiple laboratories usually report results from experiments using subsystems developed at each laboratories” (the logical negation) and it would not conflict with any part of the stimulus.

    I hope that helps at least a little. I enjoy writing these explanations, they help to solidify my mental process as well. Happy studying!
  • chinobonitochinobonito Member
    105 karma
    can u post the question here so i can attempt to read it and explain it ? i cannot find this problem, thanks!
  • brna0714brna0714 Alum Inactive ⭐
    1489 karma
    I'm not sure posting entire questions is permitted since the LSAC has to be paid royalties for their work. Maybe a mod can clarify? @"Dillon A. Wright"
  • inactiveinactive Alum Member
    edited March 2015 12637 karma
    Sorry @chinobonito , posting entire questions or even segments of any PT questions on the forums is against the rules!
    You can link the question for members who can't find it, though!

    As for PT32, it's not yet available in full on 7Sage so we don't have it.
  • Dgelf321Dgelf321 Free Trial Member
    87 karma
    Thank you @brna0714 ! It does help, and right back atcha! Happy Studying to all
  • marsasb161-1marsasb161-1 Member
    25 karma

    Why doesn't (C) work?

    Would (C) have worked if the question was a MSS or another type of question?

  • yeshesviniyeshesvini Member
    113 karma

    c doesn't work because we have only been given 2 examples to cite this- clinical trials and physics experiments.... can be alluded to some ....one can not definitely conclude however that all multi authors articles are from multiple institutions...unwarranted assumption....

    answer choices a is wrong coz of the never...b is correct as usually corresponds to most

Sign In or Register to comment.